Difference between revisions of "2022 AMC 12A Problems/Problem 24"
(→Solution) |
(→Solution) |
||
Line 19: | Line 19: | ||
By solving this recursive equation, for <math>q = 1</math> and <math>p \geq q</math>, we get | By solving this recursive equation, for <math>q = 1</math> and <math>p \geq q</math>, we get | ||
+ | <cmath> | ||
\begin{align*} | \begin{align*} | ||
N \left( p , 1 \right) | N \left( p , 1 \right) | ||
Line 26: | Line 27: | ||
& = 2^p - 1 . | & = 2^p - 1 . | ||
\end{align*} | \end{align*} | ||
+ | </cmath> | ||
For <math>q = 2</math> and <math>p \geq q</math>, we get | For <math>q = 2</math> and <math>p \geq q</math>, we get | ||
+ | <cmath> | ||
\begin{align*} | \begin{align*} | ||
N \left( p , 2 \right) | N \left( p , 2 \right) | ||
Line 39: | Line 42: | ||
& = 3^p - 2^p - p . | & = 3^p - 2^p - p . | ||
\end{align*} | \end{align*} | ||
+ | </cmath> | ||
For <math>q = 3</math> and <math>p \geq q</math>, we get | For <math>q = 3</math> and <math>p \geq q</math>, we get | ||
+ | <cmath> | ||
\begin{align*} | \begin{align*} | ||
N \left( p , 3 \right) | N \left( p , 3 \right) | ||
Line 55: | Line 60: | ||
& = 4^p - 3^p - 2^{p-1} p - \frac{3}{2} p \left( p - 1 \right) . | & = 4^p - 3^p - 2^{p-1} p - \frac{3}{2} p \left( p - 1 \right) . | ||
\end{align*} | \end{align*} | ||
+ | </cmath> | ||
For <math>q = 4</math> and <math>p = 5</math>, we get | For <math>q = 4</math> and <math>p = 5</math>, we get | ||
+ | <cmath> | ||
\begin{align*} | \begin{align*} | ||
N \left( 5 , 4 \right) | N \left( 5 , 4 \right) | ||
Line 63: | Line 70: | ||
& = \boxed{\textbf{(E) 1296}} . | & = \boxed{\textbf{(E) 1296}} . | ||
\end{align*} | \end{align*} | ||
+ | </cmath> | ||
~Steven Chen (Professor Chen Education Palace, www.professorchenedu.com) | ~Steven Chen (Professor Chen Education Palace, www.professorchenedu.com) |
Revision as of 20:43, 11 November 2022
Problem
How many strings of length 5 formed from the digits 0, 1, 2, 3, 4 are there such that for each , at least of the digits are less than ? (For example, 02214 satisfies this condition because it contains at least 1 digit less than 1, at least 2 digits less than 2, at least 3 digits less than 3, and at least 4 digits less than 4. The string 23404 does not satisfy the condition because it does not contain at least 2 digits less than 2.)
Solution
Denote by the number of -digit strings formed by using numbers , where for each , at least of the digits are less than .
We have the following recursive equation: and the boundary condition for any .
By solving this recursive equation, for and , we get
For and , we get
For and , we get
For and , we get
~Steven Chen (Professor Chen Education Palace, www.professorchenedu.com)
Video Solution
~Steven Chen (Professor Chen Education Palace, www.professorchenedu.com)